Limits 2

Calculus Level 4

lim x 0 tan 1 x sin 1 x sin 3 x = ? \large \lim_{x\to 0} \dfrac{\tan^{-1}x - \sin^{-1} x}{\sin^3x} = \, ?


The answer is -0.5.

This section requires Javascript.
You are seeing this because something didn't load right. We suggest you, (a) try refreshing the page, (b) enabling javascript if it is disabled on your browser and, finally, (c) loading the non-javascript version of this page . We're sorry about the hassle.

2 solutions

Chew-Seong Cheong
Apr 16, 2016

L = lim x 0 tan 1 x sin 1 x sin 3 x = lim x 0 g ( x ) h ( x ) Since lim x 0 g ( x ) h ( x ) = 0 0 , we can use l’H o ˆ pital’s rule. = lim x 0 1 1 + x 2 1 1 x 2 3 sin 2 x cos x Differentiate up and down once. = lim x 0 2 x ( 1 + x 2 ) 2 x ( 1 x 2 ) 3 6 sin x cos 2 x 3 sin 3 x Differentiate up and down twice. = lim x 0 6 x 2 2 ( 1 + x 2 ) 3 2 x 2 + 1 ( 1 x 2 ) 5 6 cos 3 x 12 sin 3 x 9 sin 2 x cos x Differentiate up and down thrice. = 3 6 = 1 2 = 0.5 \begin{aligned} \mathcal L & = \lim_{x \to 0} \frac{\tan^{-1}x - \sin^{-1}x}{\sin^3 x} = \lim_{x \to 0} \frac{g(x)}{h(x)} \quad \quad \small \color{#3D99F6}{\text{Since } \lim_{x \to 0} \frac{g(x)}{h(x)} = \frac{0}{0} \text{, we can use l'Hôpital's rule.}} \\ & = \lim_{x \to 0} \frac{\frac{1}{1+x^2}-\frac{1}{\sqrt{1-x^2}}}{3\sin^2 x \cos x} \quad \quad \small \color{#3D99F6}{\text{Differentiate up and down once.}} \\ & = \lim_{x \to 0} \frac{-\frac{2x}{(1+x^2)^2} - \frac{x}{(\sqrt{1-x^2})^3}}{6\sin x \cos^2 x - 3\sin^3 x} \quad \quad \small \color{#3D99F6}{\text{Differentiate up and down twice.}} \\ & = \lim_{x \to 0} \frac{\frac{6x^2-2}{(1+x^2)^3} - \frac{2x^2+1}{(\sqrt{1-x^2})^5}}{6\cos^3 x - 12 \sin^3 x - 9\sin^2 x \cos x} \quad \quad \small \color{#3D99F6}{\text{Differentiate up and down thrice.}} \\ & = \frac{-3}{6} = - \frac{1}{2} = \boxed{-0.5} \end{aligned}

Great solution (+1) only one typo in second last line, it is t h \text{h} rice just that h is missing otherwise it is cool. ʕ•ٹ•ʔ

Ashish Menon - 5 years ago

Log in to reply

Thanks. I amended the spelling mistake.

Chew-Seong Cheong - 5 years ago

Log in to reply

Haha, I was pedantic there XD

Ashish Menon - 5 years ago

Relevant wiki: Limits of Functions

Let us denote the expression by S \mathfrak{S}

S = lim x 0 t a n 1 x s i n 1 x s i n 3 x = lim x 0 ( x x 3 3 + x 5 5 ) ( x + x 3 6 + 3 x 5 40 + ) s i n 3 x \mathfrak{S} = \lim_{x\to0} \frac{tan^{-1}x-sin^{-1}x}{sin^3x} = \lim_{x\to0} \frac{(x-\frac{x^3}{3}+\frac{x^5}{5} - \cdots) - (x+\frac{x^3}{6} + \frac{3x^5}{40} + \cdots)}{sin^3x}

S = lim x 0 x x x 3 ( 1 3 + 1 6 ) + s i n 3 x = lim x 0 1 6 + 1 3 + s i n 3 x x 3 \mathfrak{S} = \lim_{x\to0} \frac{\cancel{x}-\cancel{x} -x^3(\frac{1}{3}+\frac{1}{6}) + \cdots}{sin^3x} = \lim_{x\to0} \frac{\frac{1}{6}+\frac{1}{3} + \cdots}{\frac{sin^3x}{x^3}}

All the terms when divided by x 3 x^3 the coefficients of x 3 x^3 only become x-free .

Using lim x 0 s i n x x = 0 \lim_{x\to0}\frac{sinx}{x}=0

S = 1 2 = 0.5 \mathfrak{S} = -\frac{1}{2}=\boxed{-0.5}

0 pending reports

×

Problem Loading...

Note Loading...

Set Loading...